Đến nội dung

pirate nội dung

Có 113 mục bởi pirate (Tìm giới hạn từ 27-04-2020)



Sắp theo                Sắp xếp  

#165108 Đơn giản

Đã gửi bởi pirate on 30-08-2007 - 21:39 trong Số học

Chứng minh rằng: $ a^n \vdots b^n \Leftrightarrow a \vdots b$



#165105 Mệt nhoài

Đã gửi bởi pirate on 30-08-2007 - 21:32 trong Số học

Cho $ a, b$ là 2 số nguyên dương nguyên tố cùng nhau. Chứng minh:

$ (\dfrac{a^n-b^n}{a-b},a-b)=1$ hoặc $ n$



#165104 có vẻ khó hóc búa

Đã gửi bởi pirate on 30-08-2007 - 21:23 trong Số học

Ví dụ: Với 3 số a,b,c:
$ \sum a=a+b+c$
$ \sum ab= ab+bc+ca$
Không biết nhớ có sai không?



#165102 3 biến

Đã gửi bởi pirate on 30-08-2007 - 21:16 trong Bất đẳng thức và cực trị

Mời bạn nói rõ xem ai sai mới được chứ?



#164771 tìm min

Đã gửi bởi pirate on 28-08-2007 - 09:11 trong Bất đẳng thức và cực trị

$ F=\dfrac{3}{x^2+y^2}+\dfrac{2}{xy}$
$ =\dfrac{3}{1-2xy}+\dfrac{4}{2xy}$
$ =3(\dfrac{1}{1-2xy}+\dfrac{1}{2xy})+\dfrac{1}{2xy}$
$ \geq 3\dfrac{4}{1-2xy+2xy}+\dfrac{1}{\dfrac{(x+y)^2}{2}} $
$ =3.4 + 2 =14 $
Vậy min F=14
Sau này đề nghị xem kĩ bài rồi hãy xóa nhá!



#164735 3 biến

Đã gửi bởi pirate on 27-08-2007 - 22:09 trong Bất đẳng thức và cực trị

Ta có:
$ a^2+b^2+c^2=(a-b)^2+(b-c)^2+(c-a)^2$
$ \Leftrightarrow a^2+b^2+c^2 -2(ab+bc+ca)=0$
$ \Leftrightarrow (c-a-b)^2=4ab$
$ \Leftrightarrow |c-a-b|=2\sqrt{ab} (1)$
$ \Leftrightarrow \sqrt{c}=\sqrt{a} \pm \sqrt{b}$
Tương tự:$ \sqrt{b}=\sqrt{a} \pm \sqrt{c}$ và $ \sqrt{b}=\sqrt{c} \pm \sqrt{a}$
Với 2 trường hợp "cộng" và "trừ" ta đều thu được: $ \sqrt{a}+\sqrt{b}+\sqrt{c}=0$
Vậy là sao, chẳng lẽ $ a=b=c=0$ à???
Có gì sai xin anh em bình tĩnh.



#164045 IMO 2007

Đã gửi bởi pirate on 22-08-2007 - 23:03 trong Số học

bài này không đến nỗi phải dùng đến kiến thức cấp 3 nhỉ



#164039 Giúp em với !( Toán lớp 8)

Đã gửi bởi pirate on 22-08-2007 - 22:10 trong Số học

bài 2 hình như là đề PTNK thì phải



#164033 IMO 2007

Đã gửi bởi pirate on 22-08-2007 - 21:52 trong Số học

Chứng minh rằng nếu $ (4a^2-1)^2 \vdots (4ab-1)$ thì $ a=b$.
Với $ a,b $ là số nguyên dương.



#163799 Mệnh đề tương đương

Đã gửi bởi pirate on 21-08-2007 - 15:45 trong Đại số

Biến đổi ta được 3 phương trình
$a-b=\dfrac{b-c}{bc} (1); b-c=\dfrac{c-a}{ca} (2); c-a=\dfrac{a-b}{ab}$
Thế (1),(3) vào (2) ta được:
$ (a-b)bc=\dfrac{ac(a-b)}{ab}$
$ \Leftrightarrow b=\dfrac{1}{b}$
$ \Leftrightarrow b= \pm 1 $
Tương tự $ a=b=c= \pm 1 $
Vậy $ a^2b^2c^2=1$



#163791 giúp mình với

Đã gửi bởi pirate on 21-08-2007 - 15:24 trong Bất đẳng thức và cực trị

Bất đẳng thức Bunhiacopxki:
$ (a_1b_1+a_2b_2+...+a_nb_n)^2 \leq (a_1+a_2+...+a_n)(b_1+b_2+...+b_n)$
Có thể chứng minh bằng quy nạp.
Tham khảo ở bất cứ cuốn sách về BDT nào



#163671 Dễ

Đã gửi bởi pirate on 20-08-2007 - 15:30 trong Bất đẳng thức và cực trị

Xin lỗi!!!
$\sum\limits_{}^{}\dfrac{1}{\sqrt{6(a+2)}} \leq \dfrac{1}{2\sqrt{2}} \sum\limits_{}^{}(\dfrac{1}{a+2}+\dfrac{1}{3}) \leq \dfrac{\sqrt{2}}{2}$

$ \Leftrightarrow abc+ \sum\limits_{}^{}ab -4 \geq 0$
(Bất đẳng thức cuối dễ dàng chứng minh).



#163621 Bài toán hình học

Đã gửi bởi pirate on 20-08-2007 - 09:30 trong Hình học

Chắc là ghi sai đề, theo tôi phải là góc AMN=90 độ
Gọi K là trung điểm AC.
BAKM nội tiếp suy ra ANMK nội tiếp
lại suy ra ANMK nội tiếp
Vậy góc AMN là góc vuông



#163616 Dễ

Đã gửi bởi pirate on 20-08-2007 - 08:55 trong Bất đẳng thức và cực trị

Ta có: $ a^2+4a+13 \geq 6a+12 = 6(a+2)$
$ \Rightarrow \sum\limits_{}^{}\dfrac{1}{\sqrt{a^2+4a+13}} \leq \sum\limits_{}^{}\dfrac{1}{\sqrt{6(a+2)}} \leq 3 \sqrt[3]{\dfrac{1}{sqrt{216(a+2)(b+2)(c+2)}}} \leq \dfrac{\sqrt{2}}{2}$

Ta có: $ a^2+4a+13 \geq 6a+12 = 6(a+2)$
$ \Rightarrow \sum\limits_{}^{}\dfrac{1}{\sqrt{a^2+4a+13}} \leq \sum\limits_{}^{}\dfrac{1}{\sqrt{6(a+2)}} \leq 3 \sqrt[3]{\dfrac{1}{sqrt{216(a+2)(b+2)(c+2)}}} \leq \dfrac{\sqrt{2}}{2}$

($(a+2)(b+2)(c+2)=abc+2(ab+bc+ca)+4(a+b+c)+8 \geq abc+2.3\sqrt[3]{(abc)^2}+4.3\sqrt[3]{abc}+8=27$)



#163204 Hệ Phương Trình

Đã gửi bởi pirate on 16-08-2007 - 21:05 trong Phương trình, hệ phương trình và bất phương trình

Tìm nghiệm nguyên dương x,y,z,t của hệ phương trình:
$ \left\{\begin{array}{l}a+b=3k\\ax^2+by^2=z^2\\bx^2+ay^2=t^2\end{array}\right.$
Với (3,$k$)=1.

Để giải hệ này, ta chỉ cần giải phương trình:
$ 3k(x^2+y^2)=z^2+t^2$



#160514 Đồng quy (tập 2)

Đã gửi bởi pirate on 16-07-2007 - 21:04 trong Hình học

Làm ơn giúp tôi, đang bí phần 3.



#160513 Đồng quy (tập 3)

Đã gửi bởi pirate on 16-07-2007 - 21:03 trong Hình học

Phần 2 của loạt phim đã không ăn khách như mong đợi, sau đây là một bài mới mà cũ xì:
Cho tam giác $ \triangle ABC$ nội tiếp đường tròn $(O) $. Vẽ đường tròn $ (O')$ tiếp xúc trong với (O) tại T, tiếp xúc với AB, AC, lần lượt tại E, F.
1.Chứng minh EF đồng quy với các đường phân giác của $ \triangle ABC $
2.(Hơi lạc đề một tí) Gọi K là giao điểm của EF với các đường phân giác của $ \triangle ABC$. Chứng minh $ \triangle BEK \sim \triangle KFC$
Sau bài này tôi tạm thời biệt tích một thời gian với lý do: dời nhà. Hy vọng sau khi gặp lại loạt phim các bạn sẽ đón nhận cả phần 2 và 3 thật nồng nhiệt (nghĩa là giải hết ấy mà)



#160510 Nhìn cái ... vào liền

Đã gửi bởi pirate on 16-07-2007 - 20:48 trong Phương trình, hệ phương trình và bất phương trình

Nói chung là thế này:
Nếu $x_0$ là nghiệm của đa thức P(x) thì khi phân tích đa thức P(x) thành nhân tử sẽ có thừa số $ x-x_0$



#160465 Thi tuyển sinh vào lớp 10 trường THPT Chuyên Phan Bội Châu

Đã gửi bởi pirate on 16-07-2007 - 10:02 trong Tài liệu - Đề thi

Cứ ra tiệm đĩa hỏi GEOMETRY SKETCHPAD nhé.



#160342 Bài cực trị(edited)

Đã gửi bởi pirate on 14-07-2007 - 22:15 trong Bất đẳng thức và cực trị

Đây, thưa anh!
$ P=\dfrac{x^2}{2}+\dfrac{x}{yz}+\dfrac{y^2}{2}+\dfrac{y}{xz}+\dfrac{z^2}{2}+\dfrac{z}{xy}$
$=\dfrac{x^2}{2}+\dfrac{x}{2yz}+\dfrac{x}{2yz}+\dfrac{y^2}{2}+\dfrac{y}{2xz}+\dfrac{y}{2xz}+\dfrac{z^2}{2}+\dfrac{z}{2xy}+\dfrac{z}{2xy}$
$ \geq 9 \sqrt[9]{({\dfrac{1}{2}})^9.\dfrac{x^9y^9x^9}{x^9y^9z^9}} = \dfrac{9}{2}$
Đẳng thức xảy ra khi $ x=y=z=1$
Hồi chiều bị mẹ hối nên mới viết tắt thế thôi, chứ tôi là người chủ trương "cây phải có đủ rễ- ngọn"
Chà! Đang đánh thì phát hiện ilovemoney_hic đã gởi bài lên trước mất rồi.
Kệ, nếu có thừa thì lại "xóa" nữa nhá!!
________________________________________________
ilovemoney_hic : Rất đồng tình với chủ trương của bạn "cây thì phải có đủ rễ - ngọn" <----------- câu này hay! :geq
À, mình đã tự ý sửa (chưa hỏi được ý kiến bạn)
${\dfrac{1}{2}}^9$ thành $({\dfrac{1}{2}})^9$
$\dfrac{a^9b^9c^9}{a^9b^9c^9}}$ thành $\dfrac{x^9y^9x^9}{x^9y^9z^9}}$
chắc không sao nhỉ :in
Và để cho các bạn tiện theo dõi, mình sẽ xóa bớt một trong hai lời giải.



#160335 Đồng quy

Đã gửi bởi pirate on 14-07-2007 - 21:16 trong Hình học

Cám ơn nhiều, các anh đã giúp tôi hiểu được nhiều điều cơ bản.
Vậy tại sao ta không chuyển qua "đồng quy tập 2" nhỉ???



#160333 1 bài dọa trẻ con<ngẫu hứng >

Đã gửi bởi pirate on 14-07-2007 - 21:10 trong Phương trình, hệ phương trình và bất phương trình

Không cần hoang tuan anh có nói là vô hạn căn hay không? Cái vô hạn căn đó là do tôi biến đổi, không liên quan đến giả thiết có cho hay không?



#160239 Malaysia 1999

Đã gửi bởi pirate on 14-07-2007 - 08:02 trong Phương trình, hệ phương trình và bất phương trình

Giải hệ phương trình:
$ \left\{\begin{array}{l} \sum\limits_{i=1}^{1999}x_i=1999\\\sum\limits_{i=1}^{1999}x_i^3 = \sum\limits_{i=1}^{1999}x_i^4\end{array}\right.$



#160237 Đồng quy

Đã gửi bởi pirate on 14-07-2007 - 07:50 trong Hình học

BẤm chỗ "Reply" ấy!!



#160222 Đồng quy

Đã gửi bởi pirate on 13-07-2007 - 22:18 trong Hình học

làm sao biết được các diện tích đó bằng nhau?????